Prove P(n): "Proof by Induction for (1+h)n\geq1+nh+\frac{n(n+1)}{2}h2

  • Thread starter Thread starter major_maths
  • Start date Start date
  • Tags Tags
    Induction Proof
Click For Summary
The discussion focuses on proving the inequality (1+h)^n ≥ 1 + nh + (n(n+1)/2)h^2 using mathematical induction. The initial step confirms that the base case P(0) holds true. Participants express confusion over the correct formulation of the inductive hypothesis, debating whether it should involve n(n-1) or n(n+1). A key point of clarification is that the correct term is n(n-1), as demonstrated by testing the case for n=2. The conversation emphasizes the importance of accurately applying the inductive hypothesis to complete the proof.
major_maths
Messages
30
Reaction score
0

Homework Statement


Prove that for any positive h and any integer n\geq0, (1+h)n\geq1+nh+\frac{n(n+1)}{2}h2.


Homework Equations


None.


The Attempt at a Solution


I proved that P(0) is true (1\geq1). The rest of the proof goes as follows:

Assume K\inZ (the set of integers) and P(K) is true.
Then (1+h)K\geq1+Kh+\frac{K(K-1)}{2}h2.
Then (1+h)(K+1) = (1+h)K+(1+h)1...

I can't figure out how to relate that part to the final part of P(K+1), which is 1+(K+1)h+\frac{(K+1)(K+1-1)}{2}h2.
 
Physics news on Phys.org
You have that (1+h)K+1=(1+h)K(1+h) = P(K)(1+h), so see where that goes.
 
I'm working on the same problem, to show P(k+1) I set it up the same way

but then we can use our inductive hypothesis

(1+x)^(k+1) >= (1+kx+(1/2)*k(k-1)*x^2)(1+x)

My question is, I've wrestled with the algebra for a little while now and for some reason in my notes i had P(K) set to:

1+kx+(1/2)*k(k+1)*x^2

(where the 1 in k(k+1) is positive instead of negative) I think my professor did the problem with k+1 instead of k-1. But i thought from teh inductive hypothesis the term is k(k-1) NOT k(k+1) because k+1 is what we get from P(k+1) that is what we get from the substitution?

I know there will be left over terms but i keep getting k(k-1)/2 instead of k(k+1)/2.
 
stihl29 said:
I'm working on the same problem, to show P(k+1) I set it up the same way

but then we can use our inductive hypothesis

(1+x)^(k+1) >= (1+kx+(1/2)*k(k-1)*x^2)(1+x)

My question is, I've wrestled with the algebra for a little while now and for some reason in my notes i had P(K) set to:

1+kx+(1/2)*k(k+1)*x^2

(where the 1 in k(k+1) is positive instead of negative) I think my professor did the problem with k+1 instead of k-1. But i thought from teh inductive hypothesis the term is k(k-1) NOT k(k+1) because k+1 is what we get from P(k+1) that is what we get from the substitution?

I know there will be left over terms but i keep getting k(k-1)/2 instead of k(k+1)/2.

I believe you are correct. It has to be n(n-1) instead of n(n+1).

This is seen by picking n=2, then

(1+h)^2=1+2h+h^2

and not

(1+h)^2=1+2h+3h^2
 
Question: A clock's minute hand has length 4 and its hour hand has length 3. What is the distance between the tips at the moment when it is increasing most rapidly?(Putnam Exam Question) Answer: Making assumption that both the hands moves at constant angular velocities, the answer is ## \sqrt{7} .## But don't you think this assumption is somewhat doubtful and wrong?

Similar threads

  • · Replies 15 ·
Replies
15
Views
3K
  • · Replies 7 ·
Replies
7
Views
2K
Replies
9
Views
2K
  • · Replies 23 ·
Replies
23
Views
2K
Replies
5
Views
2K
  • · Replies 5 ·
Replies
5
Views
2K
  • · Replies 6 ·
Replies
6
Views
2K
  • · Replies 3 ·
Replies
3
Views
2K
  • · Replies 4 ·
Replies
4
Views
2K
  • · Replies 3 ·
Replies
3
Views
2K